Practice One statement two arguments questions - verbal reasoning Online Quiz (set-1) For All Competitive Exams

Directions:
In making decisions about important questions, it is desirable to be able to distinguish between 'strong' arguments and 'weak' arguments. 'Strong' arguments are those which are both important and directly related to the question. 'Weak' arguments are those which are of minor importance and also may not be directly related to the question or may be related to a trivial aspect of the question. Each question below is followed by two arguments numbered I and II. You have to decide which of the arguments is a 'strong' argument and which is a 'weak' argument? Give answer
    (a) if only Argument I is strong
    (b) if only Argument II is strong
    (c) if either I or II is strong
    (d) if neither I nor II is strong
    (e) if both I and II are strong.

Q-1)  
  • Statement:
  • Should the sale of tobacco products be restricted to only a few outlets in each city/town?
  • Argument :
  • I. Yes, this will substantially reduce the consumption of tobacco products.
  • II. No, those who want to purchase tobacco products should get them at convenient locations.

(a)

(b)

(c)

(d)

(e)

Explanation:

Argument I is strong as reduced tobacco consumption is desirable.

Argument II is weak as such convenience is not desirable.


Directions:
In making decisions about important questions, it is desirable to be able to distinguish between 'strong' arguments and 'weak' arguments. 'Strong' arguments are those which are both important and directly related to the question. 'Weak' arguments are those which are of minor importance and also may not be directly related to the question or may be related to a trivial aspect of the question. Each question below is followed by two arguments numbered I and II. You have to decide which of the arguments is a 'strong' argument and which is a 'weak' argument? Give answer
    (a) if only Argument I is strong
    (b) if only Argument II is strong
    (c) if either I or II is strong
    (d) if neither I nor II is strong
    (e) if both I and II are strong.

Q-2)  
  • Statement:
  • Should the examination bodies for all university examinations permit the use of calculators?
  • Argument :
  • I. No, it is necessary for the students to know the methods of manual calculation to make their concepts clear.
  • II. Yes, manual calculations are no more required with extensive use of computers in all fields.

(a)

(b)

(c)

(d)

(e)

Explanation:

Methods of manual calculations are necessary for individuals. Hence, they should be encouraged.

Thus, Argument I is strong Argument II is also strong in its way but, since l and II contradict each other, we go for the choice (c).


Directions:
In making decisions about important questions, it is desirable to be able to distinguish between 'strong' arguments and 'weak' arguments. 'Strong' arguments are those which are both important and directly related to the question. 'Weak' arguments are those which are of minor importance and also may not be directly related to the question or may be related to a trivial aspect of the question. Each question below is followed by two arguments numbered I and II. You have to decide which of the arguments is a 'strong' argument and which is a 'weak' argument? Give answer
    (a) if only Argument I is strong
    (b) if only Argument II is strong
    (c) if either I or II is strong
    (d) if neither I nor II is strong
    (e) if both I and II are strong.

Q-3)  
  • Statement:
  • Should the women be advised not to travel alone at night in view of the increasing incidences of rape and sexual abuse?
  • Argument :
  • I. No, instead the government should take measures to control such incidences.
  • II. Yes, it is difficult to even for the police department to control such cases.

(a)

(b)

(c)

(d)

(e)

Explanation:

Argument I does not establish proper relation with the statement.

Argument II may be an opinion and has not been discussed in the given statement.

Hence, I and II both are weak.


Directions:
In making decisions about important questions, it is desirable to be able to distinguish between 'strong' arguments and 'weak' arguments. 'Strong' arguments are those which are both important and directly related to the question. 'Weak' arguments are those which are of minor importance and also may not be directly related to the question or may be related to a trivial aspect of the question. Each question below is followed by two arguments numbered I and II. You have to decide which of the arguments is a 'strong' argument and which is a 'weak' argument? Give answer
    (a) if only Argument I is strong
    (b) if only Argument II is strong
    (c) if either I or II is strong
    (d) if neither I nor II is strong
    (e) if both I and II are strong.

Q-4)  
  • Statement:
  • Should India make efforts to harness solar energy to fulfill its energy requirement?
  • Argument :
  • I. Yes, most of the energy sources used at present are exhaustible.
  • II. No, harnessing solar energy requires a lot of capital, which India lacks in.

(a)

(b)

(c)

(d)

(e)

Explanation:

Argument I is strong i.e., India should make efforts to harness solar energy to fulfill its energy requirement.

Because most of the energy sources used at present are exhaustible. Argument II is weak.


Directions:
Directions: Each question given below consists of a statement, followed by two arguments numbered I and II. You have to decide which of the arguments is a 'strong' argument and which is a 'weak' argument. Give answer
    (a) if only argument I is strong;
    (b) if only argument II is strong;
    (c) if either I or II is strong;
    (d) if neither I nor II is strong and
    (e) if both I and II are strong.

Q-5)  
  • Statement:
  • Should agriculture in rural India be mechanized?
  • Argument :
  • I. Yes. It would lead to higher production.
  • II. No. Many villagers would be left unemployed.

(a)

(b)

(c)

(d)

(e)

Explanation:

Clearly, mechanization would speed up the work and increase production. So, the argument I is strong enough.

Argument II is vague because mechanization will only eliminate wasteful employment, not create unemployment.


Directions:
Directions: Each question given below consists of a statement, followed by two arguments numbered I and II. You have to decide which of the arguments is a 'strong' argument and which is a 'weak' argument. Give answer
    (a) if only argument I is strong;
    (b) if only argument II is strong;
    (c) if either I or II is strong;
    (d) if neither I nor II is strong and
    (e) if both I and II are strong.

Q-6)  
  • Statement:
  • Should the educated unemployed youth be paid "unemployment allowance" by the Government?
  • Argument :
  • I. Yes. It will provide them some monetary help to either seek employment or to kickstart some 'self-employment' venture.
  • II. No. It will dampen their urge to do something to earn their livelihood and thus promote idleness among the unemployed youth.

(a)

(b)

(c)

(d)

(e)

Explanation:

Young people, who do not get employment due to a large number of applicants in all fields, must surely be given an allowance so that they can support themselves.

So, argument I is valid. However, such allowances would mar the spirit to work, in them and make them idle. So, argument II also holds.


Directions:
Directions: Each question given below consists of a statement, followed by two arguments numbered I and II. You have to decide which of the arguments is a 'strong' argument and which is a 'weak' argument. Give answer
    (a) if only argument I is strong;
    (b) if only argument II is strong;
    (c) if either I or II is strong;
    (d) if neither I nor II is strong and
    (e) if both I and II are strong.

Q-7)  
  • Statement:
  • Should all the practising doctors be brought under Government control so that they get a salary from the Government and treat patients free of cost?
  • Argument :
  • I. No. How can any country do such an undemocratic thing?
  • II. Yes. Despite many problems, it will certainly help minimize, if not eradicate, unethical medical practices.

(a)

(b)

(c)

(d)

(e)

Explanation:

A doctor treating a patient individually can mislead the patient into wrong and unnecessary treatment for his personal gain. So, argument II holds strong.

Also, a policy beneficial to common people cannot be termed 'undemocratic’. So, I is vague.


Directions:
Directions: Each question given below consists of a statement, followed by two arguments numbered I and II. You have to decide which of the arguments is a 'strong' argument and which is a 'weak' argument. Give answer
    (a) if only argument I is strong;
    (b) if only argument II is strong;
    (c) if either I or II is strong;
    (d) if neither I nor II is strong and
    (e) if both I and II are strong.

Q-8)  
  • Statement:
  • Should there be more than one High Courts in each state in India?
  • Argument :
  • I. No. This will be a sheer wastage of taxpayers' money.
  • II. Yes. This will help reduce the backlog of cases pending for a very long time.

(a)

(b)

(c)

(d)

(e)

Explanation:

Clearly, an increase in the number of High Courts will surely speed up the work and help to do away with the pending cases. So, argument II holds strong.

In light of this, the expenditure incurred would be ‘utilization', not 'wastage’ of money. So, argument I do not hold.


Directions:
Directions: Each question given below consists of a statement, followed by two arguments numbered I and II. You have to decide which of the arguments is a 'strong' argument and which is a 'weak' argument. Give answer
    (a) if only argument I is strong;
    (b) if only argument II is strong;
    (c) if either I or II is strong;
    (d) if neither I nor II is strong and
    (e) if both I and II are strong.

Q-9)  
  • Statement:
  • Should India give away Kashmir to Pakistan?
  • Argument :
  • I. No. Kashmir is a beautiful state. It earns a lot of foreign exchange for India.
  • II. Yes. This would help settle conflicts.

(a)

(b)

(c)

(d)

(e)

Explanation:

Clearly, India cannot part with a state that is a major foreign exchange earner to it. So, argument I holds strong.

Further, giving away a piece of land unconditionally and unreasonably is no solution to settle disputes. So, argument II is vague.


Directions:
Directions: Each question given below consists of a statement, followed by two arguments numbered I and II. You have to decide which of the arguments is a 'strong' argument and which is a 'weak' argument. Give answer
    (a) if only argument I is strong;
    (b) if only argument II is strong;
    (c) if either I or II is strong;
    (d) if neither I nor II is strong and
    (e) if both I and II are strong.

Q-10)  
  • Statement:
  • Should higher education be restricted to only those who can bear the expenditure?
  • Argument :
  • I. Yes. Higher education is very costly, hence it should not be given free.
  • II. No. There are a large number of brilliant students who cannot afford to pay and they should be given higher education.

(a)

(b)

(c)

(d)

(e)

Explanation:

For the all-round progress of the nation, all the students, especially the talented and intelligent ones, must avail of higher education, even if the government has to pay for it.

So, only argument II holds.